Difference between revisions of "2005 AMC 8 Problems/Problem 21"

m (Solution 2)
(Solution 2)
 
Line 10: Line 10:
  
 
*Note:  We are assuming that there are no degenerate triangles in this problem, and that is why we subtract two.
 
*Note:  We are assuming that there are no degenerate triangles in this problem, and that is why we subtract two.
==Solution 2==
 
'''Case 1: One vertex is on the top 3 points'''
 
 
Here, there is <math>\binom 31 =3</math> ways to choose the vertex on the top and <math>\binom 32 =3</math> ways the choose the <math>2</math> on the bottom, so there is <math>3 \cdot 3=9</math> triangles.
 
 
'''Case 2: One vertex is on the bottom 3 points'''
 
 
By symmetry, there is <math>9</math> triangles.
 
 
Otherwise, the triangle is degenerate.
 
 
Our final answer is <math>9+9=\boxed{\text{(C) 18}}</math> ~[[Ddk001]]
 
 
 
==Video solution==
 
==Video solution==
  

Latest revision as of 09:51, 17 June 2024

Problem

How many distinct triangles can be drawn using three of the dots below as vertices?

[asy]dot(origin^^(1,0)^^(2,0)^^(0,1)^^(1,1)^^(2,1));[/asy]

$\textbf{(A)}\ 9\qquad\textbf{(B)}\ 12\qquad\textbf{(C)}\ 18\qquad\textbf{(D)}\ 20\qquad\textbf{(E)}\ 24$

Solution 1

The number of ways to choose three points to make a triangle is $\binom 63 = 20$. However, two* of these are a straight line so we subtract $2$ to get $\boxed{\textbf{(C)}\ 18}$.

  • Note: We are assuming that there are no degenerate triangles in this problem, and that is why we subtract two.

Video solution

https://www.youtube.com/watch?v=XQS-KVW1O6M ~David

See Also

2005 AMC 8 (ProblemsAnswer KeyResources)
Preceded by
Problem 20
Followed by
Problem 22
1 2 3 4 5 6 7 8 9 10 11 12 13 14 15 16 17 18 19 20 21 22 23 24 25
All AJHSME/AMC 8 Problems and Solutions

The problems on this page are copyrighted by the Mathematical Association of America's American Mathematics Competitions. AMC logo.png